Last visit was: 26 Apr 2024, 22:16 It is currently 26 Apr 2024, 22:16

Close
GMAT Club Daily Prep
Thank you for using the timer - this advanced tool can estimate your performance and suggest more practice questions. We have subscribed you to Daily Prep Questions via email.

Customized
for You

we will pick new questions that match your level based on your Timer History

Track
Your Progress

every week, we’ll send you an estimated GMAT score based on your performance

Practice
Pays

we will pick new questions that match your level based on your Timer History
Not interested in getting valuable practice questions and articles delivered to your email? No problem, unsubscribe here.
Close
Request Expert Reply
Confirm Cancel
SORT BY:
Kudos
Manager
Manager
Joined: 31 Jan 2012
Posts: 60
Own Kudos [?]: 60 [1]
Given Kudos: 3
Send PM
Math Expert
Joined: 02 Sep 2009
Posts: 92948
Own Kudos [?]: 619276 [1]
Given Kudos: 81609
Send PM
Tutor
Joined: 16 Oct 2010
Posts: 14831
Own Kudos [?]: 64941 [1]
Given Kudos: 427
Location: Pune, India
Send PM
Math Expert
Joined: 02 Sep 2009
Posts: 92948
Own Kudos [?]: 619276 [1]
Given Kudos: 81609
Send PM
Re: The Discreet Charm of the DS [#permalink]
1
Kudos
Expert Reply
RastogiSarthak99 wrote:
Bunuel wrote:
SOLUTIONS:

1. Bonnie can paint a stolen car in x hours, and Clyde can paint the same car in y hours. They start working simultaneously and independently at their respective constant rates at 9:45am. If both x and y are odd integers, is x=y?

Bonnie and Clyde when working together complete the painting of the car ins \(\frac{xy}{x+y}\) hours (sum of the rates equal to the combined rate or reciprocal of total time: \(\frac{1}{x}+\frac{1}{y}=\frac{1}{T}\) --> \(T=\frac{xy}{x+y}\)). Now, if \(x=y\) then the total time would be: \(\frac{x^2}{2x}=\frac{x}{2}\), since \(x\) is odd then this time would be odd/2: 0.5 hours, 1.5 hours, 2.5 hours, ....

(1) x^2+y^2<12 --> it's possible \(x\) and \(y\) to be odd and equal to each other if \(x=y=1\) but it's also possible that \(x=1\) and \(y=3\) (or vise-versa). Not sufficient.

(2) Bonnie and Clyde complete the painting of the car at 10:30am --> they complete the job in 3/4 of an hour (45 minutes), since it's not odd/2 then \(x\) and \(y\) are not equal. Sufficient.

Answer: B.


Bunuel in Statement 2, does this mean that because t = 3/4, which is a fraction, it is not odd and hence 3/4 =x and 3/4= y?

Separate question: I was confused by the wording of St2. For a second it made it sound like Bonnie and Clyde were working together on painting, but question says independent. Which statement to trust more here?


1. From the stem we concluded that IF x = y, then the total time would be (x hours)/2 = (odd hours)/2 (since given that x is odd). (odd hours)/2 means the total time would be 0.5 hours, 1.5 hours, 2.5 hours, ... (2) says that the total time is 3/4 hours. 3/4 hours is NOT in the form of (odd hours)/2, so x does not equal to y.

2. Working simultaneously and independently, means that they work on different parts of the job. For example, one is painting the front and another the back of the car. This ensure that their works does not overlap and they do the job in fastest way possible.
Math Expert
Joined: 02 Sep 2009
Posts: 92948
Own Kudos [?]: 619276 [1]
Given Kudos: 81609
Send PM
Re: The Discreet Charm of the DS [#permalink]
1
Kudos
Expert Reply
Nsp10 wrote:
3,4,5,6,7
they are 5 consecutive nos with median and mean both same as 5 and I think the only pair with the given condition,
Am I wrong in this case ?



Bunuel wrote:
4. How many numbers of 5 consecutive positive integers is divisible by 4?

(1) The median of these numbers is odd --> the median of the set with odd number of terms is just a middle term, thus our set of 5 consecutive numbers is: {Odd, Even, Odd, Even, Odd}. Out of 2 consecutive even integers only one is a multiple of 4. Sufficient.

(2) The average (arithmetic mean) of these numbers is a prime number --> in any evenly spaced set the arithmetic mean (average) is equal to the median --> mean=median=prime. Since it's not possible that median=2=even, (in this case not all 5 numbers will be positive), then median=odd prime, and we have the same case as above. Sufficient.

Answer: D.


{5, 6, 7, 8, 9}
{9, 10, 11, 12, 13}
{11, 12, 13, 14, 15}
{15, 16, 17, 18, 19}
...
Math Expert
Joined: 02 Sep 2009
Posts: 92948
Own Kudos [?]: 619276 [1]
Given Kudos: 81609
Send PM
Re: The Discreet Charm of the DS [#permalink]
1
Kudos
Expert Reply
Roop14S wrote:
Bunuel wrote:
2. Is xy<=1/2?

(1) x^2+y^2=1. Recall that \((x-y)^2\geq{0}\) (square of any number is more than or equal to zero) --> \(x^2-2xy+y^2\geq{0}\) --> since \(x^2+y^2=1\) then: \(1-2xy\geq{0}\) --> \(xy\leq{\frac{1}{2}}\). Sufficient.

(2) x^2-y^2=0 --> \(|x|=|y|\). Clearly insufficient.

Answer: A.


Hello,
in point 1 if we take (x+y)^2 >= 0, does that work as well. As it will prove that the inequality does not hold true for the given statements.


The key is that if you look at \((x+y)^2 \ge 0\), you'll find \(xy\geq{-\frac{1}{2} }\). But this isn't really helpful. It's true that both \(xy\geq{-\frac{1}{2} }\) (from \((x+y)^2 \ge 0\)) and \(xy \le \frac{1}{2}\) (from \((x-y)^2 \ge 0\)) are correct. So, this means \(-\frac{1}{2} \leq{xy} \leq{\frac{1}{2} }\). But only the method shown in the original solution gets us the answer we want. The other way just gives us an inequality that doesn't help much.

Hope it's clear.
User avatar
Intern
Intern
Joined: 29 Jan 2012
Posts: 15
Own Kudos [?]: 7 [0]
Given Kudos: 4
Location: United Kingdom
Concentration: Finance, General Management
GMAT 1: 660 Q45 V35
GMAT 2: 720 Q47 V42
GPA: 3.53
WE:Business Development (Investment Banking)
Send PM
Re: The Discreet Charm of the DS [#permalink]
kys123 wrote:
Hey Bunuel can I ask a question for 12?

We know 6a=3b

And for statement one:

ac =6b. Can't 6b =12a

Then it becomes ac=12a ==> c=12. I know it's wrong since if a is 0 then they will be equal regardless, but can you explain why what I did was wrong?


Also,

6a = 3b = 7c

Can we say a/b= 1/2, b/c = 7/3, and a/c = 7/6
a) ac = 6b, therefore c = 6b/a
substituting this in b/c => b / (6b/a) = 7/3 => a =14, b=28, c = 12

Isnt A also sufficient? Am I ignoring something?
User avatar
Intern
Intern
Joined: 08 Jan 2012
Posts: 6
Own Kudos [?]: 5 [0]
Given Kudos: 1
Send PM
Re: The Discreet Charm of the DS [#permalink]
All,

7. A certain fruit stand sold total of 76 oranges to 19 customers. How many of them bought only one orange?
(1) None of the customers bought more than 4 oranges
(2) The difference between the number of oranges bought by any two customers is even

In respect to the question above, I assumed that any two of those 19 customer might have bought 5 & 3 oranges and hence I, marked the option insufficient. Bunnel have equated and treated the option in totally different way. I ,lack the skill to convert these sort of condition in to equation.

please can some post or point to the list of similar Word translation sentences and how to convert them in to equation. Im very new to GMAT club so please forgie me if this is the repeated posting.

Thanks,
Vids
User avatar
Manager
Manager
Joined: 07 Sep 2010
Posts: 222
Own Kudos [?]: 5230 [0]
Given Kudos: 136
Send PM
Re: The Discreet Charm of the DS [#permalink]
Hi Bunuel,

Can we solve this question using graphical approach.

Since (x-a)^2+(y-b)^2 = r^2 ----(A)

Equation 1 represents equation of circle with radius 1.

Now, if we can maximize the value of both x and y, we can get the maximum value of xy and that could let us know whether statement (1) is sufficient or not.

Here, I require your help. Intuitively, I find that x=y will maximize the equation x^2+y^2=1. But I am not sure about my reasoning as well as the Mathematics behind. Could you please look into this.

Thanks

Bunuel wrote:

Please read the thread. Solutions to ALL the questions are given on the previous pages.

2. Is xy<=1/2?

(1) x^2+y^2=1. Recall that \((x-y)^2\geq{0}\) (square of any number is more than or equal to zero) --> \(x^2-2xy+y^2\geq{0}\) --> since \(x^2+y^2=1\) then: \(1-2xy\geq{0}\) --> \(xy\leq{\frac{1}{2}}\). Sufficient.

(2) x^2-y^2=0 --> \(|x|=|y|\). Clearly insufficient.

Answer: A.
User avatar
Intern
Intern
Joined: 28 Aug 2012
Posts: 36
Own Kudos [?]: 496 [0]
Given Kudos: 105
Concentration: Operations, Marketing
GMAT 1: 510 Q36 V25
GPA: 4
WE:Information Technology (Other)
Send PM
Re: The Discreet Charm of the DS [#permalink]
Bunuel wrote:
SOLUTIONS:

1. Bonnie can paint a stolen car in x hours, and Clyde can paint the same car in y hours. They start working simultaneously and independently at their respective constant rates at 9:45am. If both x and y are odd integers, is x=y?

Bonnie and Clyde when working together complete the painting of the car ins \(\frac{xy}{x+y}\) hours (sum of the rates equal to the combined rate or reciprocal of total time: \(\frac{1}{x}+\frac{1}{y}=\frac{1}{T}\) --> \(T=\frac{xy}{x+y}\)). Now, if \(x=y\) then the total time would be: \(\frac{x^2}{2x}=\frac{x}{2}\), since \(x\) is odd then this time would be odd/2: 0.5 hours, 1.5 hours, 2.5 hours, ....

(1) x^2+y^2<12 --> it's possible \(x\) and \(y\) to be odd and equal to each other if \(x=y=1\) but it's also possible that \(x=1\) and \(y=3\) (or vise-versa). Not sufficient.

(2) Bonnie and Clyde complete the painting of the car at 10:30am --> they complete the job in 3/4 of an hour (45 minutes), since it's not odd/2 then \(x\) and \(y\) are not equal. Sufficient.

Answer: B.


Bunuel,
I did not understand the reasoning behind statement 2 being sufficient. Line marked in red is my doubt. what does it mean ?
Math Expert
Joined: 02 Sep 2009
Posts: 92948
Own Kudos [?]: 619276 [0]
Given Kudos: 81609
Send PM
Re: The Discreet Charm of the DS [#permalink]
Expert Reply
thinktank wrote:
Bunuel wrote:
SOLUTIONS:

1. Bonnie can paint a stolen car in x hours, and Clyde can paint the same car in y hours. They start working simultaneously and independently at their respective constant rates at 9:45am. If both x and y are odd integers, is x=y?

Bonnie and Clyde when working together complete the painting of the car ins \(\frac{xy}{x+y}\) hours (sum of the rates equal to the combined rate or reciprocal of total time: \(\frac{1}{x}+\frac{1}{y}=\frac{1}{T}\) --> \(T=\frac{xy}{x+y}\)). Now, if \(x=y\) then the total time would be: \(\frac{x^2}{2x}=\frac{x}{2}\), since \(x\) is odd then this time would be odd/2: 0.5 hours, 1.5 hours, 2.5 hours, ....

(1) x^2+y^2<12 --> it's possible \(x\) and \(y\) to be odd and equal to each other if \(x=y=1\) but it's also possible that \(x=1\) and \(y=3\) (or vise-versa). Not sufficient.

(2) Bonnie and Clyde complete the painting of the car at 10:30am --> they complete the job in 3/4 of an hour (45 minutes), since it's not odd/2 then \(x\) and \(y\) are not equal. Sufficient.

Answer: B.


Bunuel,
I did not understand the reasoning behind statement 2 being sufficient. Line marked in red is my doubt. what does it mean ?


From the stem we got that if \(x=y\) then the total time would be: \(\frac{x^2}{2x}=\frac{x}{2}\), since \(x\) is odd then this time would be odd/2: 0.5 hours (1/2 hours = 0.5 hours), 1.5 hours (3/2 hours = 1.5 hours), 2.5 hours (5/2 hours = 2.5 hours), 3.5 hours (7/2 hours = 3.5 hours), 4.5 hours (9/2 hours = 4.5 hours), ....

Now, from the second statement we got that they complete the job in 0.75 hours, since the total time (0.75 hours) is NOT odd/2 (0.5 hours, 1.5 hours, 2.5 hours, 3.5 hours, 4.5 hours, ....), then \(x\) and \(y\) are not equal.

Hope it's clear.
avatar
Intern
Intern
Joined: 26 Aug 2012
Posts: 7
Own Kudos [?]: 49 [0]
Given Kudos: 148
Concentration: Finance, International Business
GMAT 1: 710 Q48 V38
GPA: 3.01
WE:Analyst (Investment Banking)
Send PM
Re: The Discreet Charm of the DS [#permalink]
Bunuel wrote:
12. If 6a=3b=7c, what is the value of a+b+c?

Given: \(6a=3b=7c\) --> least common multiple of 6, 3, and 7 is 42 hence we ca write: \(6a=3b=7c=42x\), for some number \(x\) --> \(a=7x\), \(b=14x\) and \(c=6x\).

(1) ac=6b --> \(7x*6x=6*14x\) --> \(x^2=2x\) --> \(x=0\) or \(x=2\). Not sufficient.

(2) 5b=8a+4c --> \(5*14x=8*7x+4*14x\) --> \(70x=80x\) --> \(10x=0\) --> \(x=0\) --> \(a=b=c=0\) --> \(a+b+c=0\). Sufficient.

Answer: B.


Hi Bunuel,

In the 1st condition if we take ac = 6b and hence multiply the the given expression 6a=3b=7c by 2 we get 12a=6b=14c.

Now substituting 12a=ac=14c

I get 12a=ac --> c = 12 and similarly a = 14 and hence b= 28.

Can you please point where am I going wrong?

Regards
Math Expert
Joined: 02 Sep 2009
Posts: 92948
Own Kudos [?]: 619276 [0]
Given Kudos: 81609
Send PM
Re: The Discreet Charm of the DS [#permalink]
Expert Reply
Spaniard wrote:
Bunuel wrote:
12. If 6a=3b=7c, what is the value of a+b+c?

Given: \(6a=3b=7c\) --> least common multiple of 6, 3, and 7 is 42 hence we ca write: \(6a=3b=7c=42x\), for some number \(x\) --> \(a=7x\), \(b=14x\) and \(c=6x\).

(1) ac=6b --> \(7x*6x=6*14x\) --> \(x^2=2x\) --> \(x=0\) or \(x=2\). Not sufficient.

(2) 5b=8a+4c --> \(5*14x=8*7x+4*14x\) --> \(70x=80x\) --> \(10x=0\) --> \(x=0\) --> \(a=b=c=0\) --> \(a+b+c=0\). Sufficient.

Answer: B.


Hi Bunuel,

In the 1st condition if we take ac = 6b and hence multiply the the given expression 6a=3b=7c by 2 we get 12a=6b=14c.

Now substituting 12a=ac=14c

I get 12a=ac --> c = 12 and similarly a = 14 and hence b= 28.

Can you please point where am I going wrong?

Regards


ac=12a (here you cannot reduce by a and write c=12 as you exclude possibility of a=0) --> a(c-12)=0 --> either a=0 OR c=12. So, we get either a=b=c=0 or a=14, b=28 and c=12.

Does this make sense?
Manager
Manager
Joined: 10 Mar 2014
Posts: 137
Own Kudos [?]: 675 [0]
Given Kudos: 13
Send PM
Re: The Discreet Charm of the DS [#permalink]
Bunuel wrote:
8. If x=0.abcd, where a, b, c and d are digits from 0 to 9, inclusive, is x>7/9?

First of all 7/9 is a recurring decimal =0.77(7). For more on converting Converting Decimals to Fractions see: math-number-theory-88376.html

(1) a+b>14 --> the least value of a is 6 (6+9=15>14), so in this case x=0.69d<0.77(7) but a=7 and b=9 is also possible, and in this case x=0.79d>0.77(7). Not sufficient.

(2) a-c>6 --> the least value of a is 7 (7-0=7>6), but we don't know the value of b. Not sufficient.

(1)+(2) The least value of a is 7 and in this case from (1) least value of b is 8 (7+8=15>14), hence the least value of x=0.78d>0.77(7). Sufficient.

Answer: C.


Hi Bunnel,

One question here.

in st2 i can consider 8-0 also then why we are considering only least value. in st1 we are considering least value as well as other value. why this is different in st2?

Thanks
Math Expert
Joined: 02 Sep 2009
Posts: 92948
Own Kudos [?]: 619276 [0]
Given Kudos: 81609
Send PM
Re: The Discreet Charm of the DS [#permalink]
Expert Reply
PathFinder007 wrote:
Bunuel wrote:
8. If x=0.abcd, where a, b, c and d are digits from 0 to 9, inclusive, is x>7/9?

First of all 7/9 is a recurring decimal =0.77(7). For more on converting Converting Decimals to Fractions see: math-number-theory-88376.html

(1) a+b>14 --> the least value of a is 6 (6+9=15>14), so in this case x=0.69d<0.77(7) but a=7 and b=9 is also possible, and in this case x=0.79d>0.77(7). Not sufficient.

(2) a-c>6 --> the least value of a is 7 (7-0=7>6), but we don't know the value of b. Not sufficient.

(1)+(2) The least value of a is 7 and in this case from (1) least value of b is 8 (7+8=15>14), hence the least value of x=0.78d>0.77(7). Sufficient.

Answer: C.


Hi Bunnel,

One question here.

in st2 i can consider 8-0 also then why we are considering only least value. in st1 we are considering least value as well as other value. why this is different in st2?

Thanks


Because considering the least value of a (7), while knowing nothing about b is enough to say that the second statement is not sufficient. If b=8, then the answer is yes but if b is 0, then the answer is no.
avatar
Intern
Intern
Joined: 30 Apr 2014
Posts: 3
Own Kudos [?]: [0]
Given Kudos: 1
Send PM
Re: The Discreet Charm of the DS [#permalink]
Hi Bunuel,
Referring to below:
4. How many numbers of 5 consecutive positive integers is divisible by 4?
(1) The median of these numbers is odd
(2) The average (arithmetic mean) of these numbers is a prime number

The average (arithmetic mean) of these numbers is a prime number --> in any evenly spaced set the arithmetic mean (average) is equal to the median --> mean=median=prime. Since it's not possible that median=2=even, (in this case not all 5 numbers will be positive), then median=odd prime, and we have the same case as above. Sufficient.

I didn't quite get the explanation on the second statement. Do you mind explaining with some numbers?

Thanks,
Mitesh
Math Expert
Joined: 02 Sep 2009
Posts: 92948
Own Kudos [?]: 619276 [0]
Given Kudos: 81609
Send PM
Re: The Discreet Charm of the DS [#permalink]
Expert Reply
mrvora wrote:
Hi Bunuel,
Referring to below:
4. How many numbers of 5 consecutive positive integers is divisible by 4?
(1) The median of these numbers is odd
(2) The average (arithmetic mean) of these numbers is a prime number

The average (arithmetic mean) of these numbers is a prime number --> in any evenly spaced set the arithmetic mean (average) is equal to the median --> mean=median=prime. Since it's not possible that median=2=even, (in this case not all 5 numbers will be positive), then median=odd prime, and we have the same case as above. Sufficient.

I didn't quite get the explanation on the second statement. Do you mind explaining with some numbers?

Thanks,
Mitesh


If mean=median=prime=2, then the 5 consecutive integers would be {0, 1, 2, 3, 4}. But this set is not possible since we are told that the set consists of 5 positive integers and 0 is neither positive nor negative integer.

Hence, mean=median=odd prime --> {Odd, Even, Odd prime, Even, Odd}. So, the set includes 2 consecutive even numbers. Out of 2 consecutive even integers only one is a multiple of 4.

Hope it's clear.
Manager
Manager
Joined: 09 May 2016
Posts: 54
Own Kudos [?]: 52 [0]
Given Kudos: 12
Location: United States (NY)
Concentration: Finance, General Management
GMAT 1: 710 Q49 V38
WE:Information Technology (Consulting)
Send PM
Re: The Discreet Charm of the DS [#permalink]
Bunuel wrote:
5. What is the value of integer x?

(1) 2x^2+9<9x --> factor qudratics: \((x-\frac{3}{2})(x-3)<0\) --> roots are \(\frac{3}{2}\) and 3 --> "<" sign indicates that the solution lies between the roots: \(1.5<x<3\) --> since there only integer in this range is 2 then \(x=2\). Sufficient.

(2) |x+10|=2x+8 --> LHS is an absolute value, which is always non negative, hence RHS must also be non-negative: \(2x+8\geq{0}\) --> \(x\geq{-4}\), for this range \(x+10\) is positive hence \(|x+10|=x+10\) --> \(x+10=2x+8\) --> \(x=2\). Sufficient.

Answer: D.



Hi Bunuel, a small doubt instead of \(2x+8\geq{0}\) --> \(x\geq{-4}\) this logic if we take |x+10| to be both positive and negative we get 2 vale by solving equation x=2 and x=-6. however by putting values back in eqtn we can see that only for x=2 equation is satisfying. Is this correct approach?
Math Expert
Joined: 02 Sep 2009
Posts: 92948
Own Kudos [?]: 619276 [0]
Given Kudos: 81609
Send PM
Re: The Discreet Charm of the DS [#permalink]
Expert Reply
KARISHMA315 wrote:
Bunuel wrote:
5. What is the value of integer x?

(1) 2x^2+9<9x --> factor qudratics: \((x-\frac{3}{2})(x-3)<0\) --> roots are \(\frac{3}{2}\) and 3 --> "<" sign indicates that the solution lies between the roots: \(1.5<x<3\) --> since there only integer in this range is 2 then \(x=2\). Sufficient.

(2) |x+10|=2x+8 --> LHS is an absolute value, which is always non negative, hence RHS must also be non-negative: \(2x+8\geq{0}\) --> \(x\geq{-4}\), for this range \(x+10\) is positive hence \(|x+10|=x+10\) --> \(x+10=2x+8\) --> \(x=2\). Sufficient.

Answer: D.



Hi Bunuel, a small doubt instead of \(2x+8\geq{0}\) --> \(x\geq{-4}\) this logic if we take |x+10| to be both positive and negative we get 2 vale by solving equation x=2 and x=-6. however by putting values back in eqtn we can see that only for x=2 equation is satisfying. Is this correct approach?


Yes, that's also a correct way of solving. Good thing you did is that you did not forget to test both values after you got them.
Intern
Intern
Joined: 25 Jan 2020
Posts: 15
Own Kudos [?]: 24 [0]
Given Kudos: 8
Location: India
Schools: IIMC PGPEX'22
GMAT 1: 730 Q50 V39
GPA: 4
WE:Sales (Computer Software)
Send PM
Re: The Discreet Charm of the DS [#permalink]
Bunuel
In
2. Is xy<=1/2?
(1) x^2+y^2=1
(2) x^2-y^2=0

If we take (X+Y)^2 instead of (X-Y)^2 for analyzing statement 1, why we get two different answers.
Also, the detailed explanation link is not working perhaps.
GMAT Club Bot
Re: The Discreet Charm of the DS [#permalink]
   1   2   3   
Moderator:
Math Expert
92948 posts

Powered by phpBB © phpBB Group | Emoji artwork provided by EmojiOne